Find u.v with Given Constraints: ||u+v||=1 and ||u-v||=5

Click For Summary
SUMMARY

The discussion focuses on finding vectors u and v that satisfy the constraints ||u+v||=1 and ||u-v||=5. The participants conclude that the vectors u and v are not orthogonal, making the Pythagorean Theorem inapplicable. They explore squaring the equations and substituting values but realize that multiple solutions exist due to the rotational symmetry of the vectors. Examples provided include u=3, v=-2 for R^1 and u=(3,0), v=(-2,0) for R^2, indicating that any solution is acceptable as long as it meets the given constraints.

PREREQUISITES
  • Understanding of vector norms and properties
  • Familiarity with the Pythagorean Theorem in vector spaces
  • Basic knowledge of vector rotation and dimensionality (R^1, R^2, R^3)
  • Experience with algebraic manipulation of equations
NEXT STEPS
  • Explore vector norms and their properties in depth
  • Learn about vector rotation and its implications in R^2 and R^3
  • Study the concept of linear combinations of vectors
  • Investigate systems of equations involving multiple variables
USEFUL FOR

Mathematicians, physics students, and anyone interested in vector analysis and problem-solving in linear algebra.

kuahji
Messages
390
Reaction score
2
Find u.v given that ||u+v||=1 & ||u-v||=5.

The first thing I did was drawn a simple picture, it became apparent that u & v wouldn't be orthogonal. So then the Pythagorean Theorem wouldn't work. Next I moved on to squaring both sides
||u+v||^{2}=||u||^{2}+2(u.v)+||u||^{2}
However here again, I didn't seem to be getting anywhere because I can't do anything with the middle term. I also tried squaring the other equation, solving for ||u|| & substituting it into the other equation. But that still left me with two variables in one equations. So I'm kinda lost about what to actually do in this problem.
 
Physics news on Phys.org
There isn't any unique solution. If you find a u and v that satisfy that then you can always rotate u and v by any angle and they will still satisfy that relation. Are you just supposed to find ANY u and v?
 
Yes any solution. We had a similar one earlier that had infinitely many solutions. But it was specified that it was R^2 or R^3. Here it does not say. We haven't learned about rotations yet, that is the next section.
 
How about R^1? u=3, v=-2. Is that good enough? If it's R^2, u=(3,0), v=(-2,0). Etc.
 
Question: A clock's minute hand has length 4 and its hour hand has length 3. What is the distance between the tips at the moment when it is increasing most rapidly?(Putnam Exam Question) Answer: Making assumption that both the hands moves at constant angular velocities, the answer is ## \sqrt{7} .## But don't you think this assumption is somewhat doubtful and wrong?

Similar threads

  • · Replies 8 ·
Replies
8
Views
3K
Replies
1
Views
1K
  • · Replies 4 ·
Replies
4
Views
2K
Replies
12
Views
2K
  • · Replies 5 ·
Replies
5
Views
2K
  • · Replies 5 ·
Replies
5
Views
2K
  • · Replies 2 ·
Replies
2
Views
1K
  • · Replies 6 ·
Replies
6
Views
2K
  • · Replies 5 ·
Replies
5
Views
7K
  • · Replies 10 ·
Replies
10
Views
3K